heureka

avatar
यूजर का नामheureka
स्कोर26376
Membership
Stats
सवाल 17
जवाब 5678

 #3
avatar+26376 
+2

Find the sum of the infinite series
\(1+2\left(\dfrac{1}{1998}\right)+3\left(\dfrac{1}{1998}\right)^2+4\left(\dfrac{1}{1998}\right)^3+\cdots\).

 

\(\text{Let $r=\dfrac{1}{1998} $}\)

 

\(\begin{array}{|rcll|} \hline s&=& \mathbf{1+2\left(\dfrac{1}{1998}\right)+3\left(\dfrac{1}{1998}\right)^2+4\left(\dfrac{1}{1998}\right)^3+\cdots} \\ s&=& \mathbf{1+2r +3r^2+4r^3+\cdots + nr^{n-1} + \ldots} \\ \hline \end{array}\)

 

\(\begin{array}{|rcrl|} \hline s_n &=& 1+&2r +3r^2+4r^3+\cdots + nr^{n-1} \quad |\quad \cdot r \\ rs_n &=& & 1r +2r^2+3r^3+\cdots + (n-1)r^{n-1}+nr^n \\ \hline s_n-rs_n &=& 1+&r+r^2+r^3+\ldots + r^{n-1} -nr^n \\ \hline \end{array}\\ \begin{array}{|rclrcrl|} \hline s_n(1-r) &=& \underbrace{1+r+r^2+r^3+\ldots + r^{n-1}}_{=S_n(\text{ geometric progression})} -nr^n \\ s_n(1-r) &=& S_n -nr^n & S_n &=& 1+&r+r^2+r^3+\ldots + r^{n-1} \quad | \quad \cdot r\\ & & & rS_n &=& &r+r^2+r^3+\ldots + r^{n-1}+r^n \\ \hline & & & S_n-rS_n &=& 1- &r^n \\ & & & S_n(1-r) &=& 1- &r^n \\ & & & S_n &=& \dfrac{1- r^n}{1-r} \\ s_n(1-r) &=& \dfrac{1- r^n}{1-r} -nr^n \\ s_n &=& \dfrac{1- r^n}{(1-r)^2} - \dfrac{nr^n}{1-r} \\ s &=& \lim \limits_{n\to \infty} s_n \\ s &=& \lim \limits_{n\to \infty} \dfrac{1- r^n}{(1-r)^2} - \dfrac{nr^n}{1-r} \\ && \boxed{\lim \limits_{n\to \infty} r^n = \lim \limits_{n\to \infty} \left(\dfrac{1}{1998}\right)^n = 0} \\ s &=& \dfrac{1- 0}{(1-r)^2} - 0 \\ s &=& \dfrac{1}{(1-r)^2} \\ s &=& \dfrac{1}{\left(1-\dfrac{1}{1998}\right)^2} \\ s &=& \left(\dfrac{1998}{1997}\right)^2 \\ \\ s &=& \dfrac{1998^2}{1997^2} \\\\ s &=&\mathbf{ \dfrac{3992004}{3988009}} \\\\ \mathbf{s} &=& \mathbf{1.00100175300507095\ldots...} \\ \hline \end{array}\)

 

laugh

18 Mei 2020
 #2
avatar+26376 
+2

Find the sum of the infinite series

\(1+2\left(\dfrac{1}{1998}\right)+3\left(\dfrac{1}{1998}\right)^2+4\left(\dfrac{1}{1998}\right)^3+\cdots\)

 

\(\small{ \begin{array}{rclllllllllc} & & \mathbf{1}&\mathbf{+} & \mathbf{2\left(\dfrac{1}{1998}\right)} &\mathbf{+} & \mathbf{3\left(\dfrac{1}{1998}\right)^2} &\mathbf{+} &\mathbf{ 4\left(\dfrac{1}{1998}\right)^3} & \mathbf{+}&\mathbf{\cdots} \\\\ & & & & & & & & & & &\mathbf{\text{sum }=\dfrac{a}{1-r}} \\ &=& 1&+ & 1\left(\dfrac{1}{1998}\right)^1 &+ & 1\left(\dfrac{1}{1998}\right)^2 &+ & 1\left(\dfrac{1}{1998}\right)^3 &+&\cdots & \dfrac{1}{1-\dfrac{1}{1998}} \\\\ & & &+ & 1\left(\dfrac{1}{1998}\right)^1 &+ & 1\left(\dfrac{1}{1998}\right)^2 &+ & 1\left(\dfrac{1}{1998}\right)^3 &+&\cdots & \dfrac{\left(\dfrac{1}{1998}\right)^1}{1-\dfrac{1}{1998}} \\\\ & & & & &+ & 1\left(\dfrac{1}{1998}\right)^2 &+ & 1\left(\dfrac{1}{1998}\right)^3 &+&\cdots & \dfrac{\left(\dfrac{1}{1998}\right)^2}{1-\dfrac{1}{1998}} \\\\ & & & & & & & + & 1\left(\dfrac{1}{1998}\right)^3 &+&\cdots & \dfrac{\left(\dfrac{1}{1998}\right)^3}{1-\dfrac{1}{1998}} \\\\ & & & & & & & & & +&\ldots \\ \end{array} }\)

 

\(\begin{array}{|rcll|} \hline &&\mathbf{ 1+2\left(\dfrac{1}{1998}\right)+3\left(\dfrac{1}{1998}\right)^2+4\left(\dfrac{1}{1998}\right)^3+\cdots } \\\\ &=& \dfrac{1}{1-\dfrac{1}{1998}} + \dfrac{\left(\dfrac{1}{1998}\right)^1}{1-\dfrac{1}{1998}} + \dfrac{\left(\dfrac{1}{1998}\right)^2}{1-\dfrac{1}{1998}} + \dfrac{\left(\dfrac{1}{1998}\right)^3}{1-\dfrac{1}{1998}} +\cdots \\\\ &=& \dfrac{1}{1-\dfrac{1}{1998}} \left( 1+ \left(\dfrac{1}{1998}\right)^1+\left(\dfrac{1}{1998}\right)^2+\left(\dfrac{1}{1998}\right)^3 +\cdots \right) \\\\ &=& \dfrac{1}{\dfrac{1998-1}{1998}} \left( 1+ \left(\dfrac{1}{1998}\right)^1+\left(\dfrac{1}{1998}\right)^2+\left(\dfrac{1}{1998}\right)^3 +\cdots \right) \\\\ &=& \dfrac{1998}{1997} \left( 1+ \left(\dfrac{1}{1998}\right)^1+\left(\dfrac{1}{1998}\right)^2+\left(\dfrac{1}{1998}\right)^3 +\cdots \right) \\\\ &=& \dfrac{1998}{1997} \left( \dfrac{1}{1-\dfrac{1}{1998}} \right) \\\\ &=& \dfrac{1998}{1997} \left( \dfrac{1}{\dfrac{1998-1}{1998}} \right) \\\\ &=& \dfrac{1998}{1997}\times \dfrac{1998}{1997} \\\\ &=& \dfrac{1998^2}{1997^2} \\\\ &=&\mathbf{ \dfrac{3992004}{3988009}} \\ \hline \end{array}\)

 

laugh

18 Mei 2020
 #2
avatar+26376 
+1

If the 4014th term of a geometric sequence of non-negative numbers is 135,

and the 14th term is 375,

what is the 2014th term?

 

\(\begin{array}{|rcll|} \hline i < j < k \\ a_i &=& ar^{i-1} \\ a_j &=& ar^{j-1} \\ a_k &=& ar^{k-1} \\ \hline \dfrac{a_k}{a_i} &=& r^{(k-1)-(i-1)} \\ \mathbf{\dfrac{a_k}{a_i}} &=& \mathbf{r^{k-i}} \\\\ \dfrac{a_k}{a_j} &=& r^{(k-1)-(j-1)} \\ \mathbf{\dfrac{a_k}{a_j}} &=& \mathbf{r^{k-j}} \\ \hline r= \left(\dfrac{a_k}{a_i}\right)^{\frac{1}{k-i}} &=& \left(\dfrac{a_k}{a_j}\right)^{\frac{1}{k-j}} \\\\ a_k^{\frac{1}{k-i}} a_j^{\frac{1}{k-j}} &=& a_k^{\frac{1}{k-j}} a_i^{\frac{1}{k-i}} \\\\ a_j^{\frac{1}{k-j}} &=& a_k^{\frac{1}{k-j}-\frac{1}{k-i}} a_i^{\frac{1}{k-i}} \\\\ a_j^{\frac{1}{k-j}} &=& a_k^{\frac{k-i-k+j}{(k-j)(k-i)}} a_i^{\frac{1}{k-i}} \\\\ a_j^{\frac{1}{k-j}} &=& a_k^{\frac{j-i}{(k-j)(k-i)}} a_i^{\frac{1}{k-i}} \\\\ \mathbf{a_j} &=& \mathbf{a_k^{\frac{j-i}{k-i}} a_i^{\frac{k-j}{k-i}} } \\ \hline \end{array}\)

 

\(\begin{array}{|rcll|} \hline \mathbf{a_j} &=& \mathbf{a_k^{\frac{j-i}{k-i}} a_i^{\frac{k-j}{k-i}} } \\\\ && \boxed{a_{14} =375=a_i\quad i = 14\\ a_{2014}=a_j \quad j=2014\\ a_{4014}=135=a_k\quad k=4014} \\\\ a_{2014} &=& 135^{\frac{2014-14}{4014-14}} 375^{\frac{4014-2014}{4014-14}} \\\\ a_{2014} &=& 135^{\frac{2000}{4000}} 375^{\frac{2000}{4000}} \\\\ a_{2014} &=& 135^{\frac{1}{2}} 375^{\frac{1}{2}} \\\\ a_{2014} &=& \sqrt{135*375} \\ a_{2014} &=& \sqrt{50625} \\ \mathbf{a_{2014}} &=& \mathbf{225} \\ \hline \end{array}\)

 

The 2014th term is 225

 

laugh

16 Mei 2020
 #1
avatar+26376 
+1

The quadratic polynomial p(x) leaves a remainder of 1 on division by x - 1 or x - 2.  
The product of the roots of p(x) is 1.  
What is the sum of the roots of p(x)?

 

The quadratic polynomial is \( p(x)=ax^2+bx+c\)

\(\begin{array}{|lrcll|} \hline & p(x) &=& q_1(x)(x-1)+1 \quad | \quad p(x)=ax^2+bx+c \\ x=1: & ax^2+bx+c &=& q_1(x)(x-1)+1 \\ & a+b+c &=& q_1(x)\cdot 0 +1 \\ &\mathbf{ a+b+c} &=& \mathbf{1} \qquad (1) \\ \hline & p(x) &=& q_2(x)(x-2)+1 \quad | \quad p(x)=ax^2+bx+c \\ x=2: & p(x)=ax^2+bx+c &=& q_2(x)(x-2)+1 \\ & 4a+2b+c &=& q_2(x)\cdot 0+1 \\ & \mathbf{4a+2b+c} &=& 1 \qquad (2) \\ \hline \end{array}\)

 

We have roots \(r_1\) and \(r_2\)

\(\begin{array}{|rcll|} \hline p(x)=ax^2+bx+c &=& 0 \\ ax^2+bx+c &=& 0 \quad | \quad : a \\ x^2+\underbrace{\frac{b}{a}}_{=-(r_1+r_2)}x+\underbrace{\frac{c}{a}}_{=r_1r_2} &=& 0 \\ \hline r_1r_2= 1 &=& \frac{c}{a} \\ 1 &=& \frac{c}{a} \\ \mathbf{c} &=& \mathbf{a} \\ \hline -(r_1+r_2) &=& \frac{b}{a} \\ \mathbf{r_1+r_2} &=& \mathbf{-\frac{b}{a}} \\ \hline \end{array}\)

 

 

\(\begin{array}{|lrcll|} \hline (1): &\mathbf{ a+b+c} &=& \mathbf{1} \quad | \quad c= a \\ & a+b+a &=& 1 \\ & 2a+b &=& 1 \\ & \mathbf{b} &=& \mathbf{1-2a} \qquad (3) \\ \\ (2): & \mathbf{4a+2b+c} &=& \mathbf{1} \quad | \quad c= a \\ & 4a+2b+a &=& 1 \\ & 5a+2b &=& 1 \quad | \quad \mathbf{b=1-2a} \\ & 5a+2(1-2a) &=& 1 \\ & 5a+2-4a &=& 1 \\ & a+2 &=& 1 \\ & a &=& 1-2 \\ & \mathbf{ a } &=& \mathbf{-1} \\\\ (3): & \mathbf{b} &=& \mathbf{1-2a} \quad | \quad \mathbf{ a =-1} \\ & b &=& 1-2(-1) \\ & b &=& 1+2 \\ & \mathbf{ b } &=& \mathbf{3} \\\\ & \mathbf{c} &=& \mathbf{a} \quad | \quad \mathbf{ a =-1} \\ & \mathbf{c} &=& \mathbf{-1} \\ \hline \end{array}\)

 

\(\begin{array}{|rcll|} \hline \mathbf{r_1+r_2} &=& \mathbf{-\frac{b}{a}}\quad | \quad b=3,\ a=-1 \\ r_1+r_2 &=& -\frac{3}{-1} \\ \mathbf{r_1+r_2} &=& \mathbf{3} \\ \hline \end{array}\)

 

The sum of the roots of \(p(x)= -x^2+3x-1\) is 3

 

laugh

15 Mei 2020